Как показать, что ψ¯γμψψ¯γμψ\bar\psi\gamma^\mu\psi спинора Дирака ψψ\psi преобразуется как вектор?

Это часть 2 упражнения II.1.1 QFT Зи в двух словах ( здесь часть 1 ).

Вот что у меня есть:

ψ ¯ γ λ ψ ψ ¯ γ λ ψ "=" ψ γ 0 γ λ ψ "=" ( С ( Λ ) ψ ) γ 0 γ λ ( С ( Λ ) ψ ) "=" ψ С ( Λ ) γ 0 γ λ С ( Λ ) ψ (1) "=" ψ е я 4 ю мю ν о мю ν γ 0 γ λ е я 4 ю мю ν о мю ν ψ "=" ψ ( 1 + я 4 ю мю ν ( о мю ν ) + О ( ю мю ν 2 ) ) γ 0 γ λ ( 1 я 4 ю мю ν о мю ν + О ( ю мю ν 2 ) ) ψ "=" ψ γ 0 ( γ λ + я 4 ю мю ν [ [ γ мю , γ ν ] , γ λ ] + О ( ю мю ν 2 ) ) ψ "=" ψ γ 0 ( γ λ + 1 2 ю мю ν [ о мю ν , γ λ ] + О ( ю мю ν 2 ) ) ψ

Два вопроса:

  • Как именно последняя строка доказывает, что ψ ¯ γ λ ψ преобразуется как вектор при преобразованиях Лоренца? Для меня это определенно выглядит как векторное преобразование из-за коммутатора между генераторами Лоренца и «векторными компонентами». γ мю , но как я могу доказать это количественно?
  • Тогда более общий вопрос: как я могу избавиться от О ( ю мю ν 2 ) ? Я знаю, что этим можно пренебречь, поскольку мы рассматриваем только бесконечно малые преобразования, поэтому поведение преобразования определяется только членами первого порядка. Но какой математически строгий способ избавиться от них? Я просто пишу подпишите и оставьте их справа от ? Мне это кажется неправильным, потому что при работе с расширениями не подразумевает строгого равенства, а просто подразумевает «равенство до определенного порядка».
Спасибо, что задали эти вопросы, в настоящее время я читаю эту книгу, и, возможно, здесь будет применяться физический прагматизм относительно: выше (разве методы типа отсечения не применяются в других расширениях?), но желаю удачи получить ответ

Ответы (1)

Будет проще, если вы воспользуетесь результатом из части 1. Тогда вам также не придется разбираться с О ( ю 2 ) (см. мой ответ на ваш другой вопрос).

В вашем расчете вы преобразовали ψ ¯ и ψ , но нет γ λ . Это правильно, как я покажу в конце, но я возьму другую точку зрения, которая здесь действительно полезна: γ λ это объект с одним индексом Лоренца λ и два фермионных индекса, поэтому он должен преобразовываться как

γ λ "=" Λ λ ν ( С γ ν С 1 )
(согласно общим правилам преобразования тензоров).

Потому что мы уже знаем из части 1, что ψ ¯ ψ ¯ С 1 , сразу получаем

ψ ¯ γ λ ψ Λ λ ν ψ ¯ С 1 С γ ν С 1 С ψ "=" Λ λ ν ψ ¯ γ ν ψ ,
это ожидаемое поведение преобразования вектора Лоренца.


То, что вы сделали, конечно, тоже правильно, но не хватает одного ингредиента. С

Λ λ ν ( С γ ν С 1 ) "=" γ λ ,
в γ матрицы на самом деле не трансформируются. Доказать это сложнее (хотя и не слишком сложно). Я не читал книгу Зи, но я предполагаю, что он где-то доказывает это?

(Вы можете начать с написания ψ ¯ γ λ ψ ψ ¯ С 1 γ λ С ψ а затем использовать это тождество, чтобы получить тот же результат.)

Спасибо, но что такое фермионный индекс и почему гамма-матрицы преобразуются таким образом (ваше первое уравнение)? Любое объяснение или ссылка на книгу приветствуются.
Другое дело: по общим правилам, как трансформируются тензоры : я не знал, что гамма-матрицы - это тензоры? Почему правила тензорного преобразования должны применяться к гаммам?
Для каждого мю , γ мю это матрица, так что гамма на самом деле является объектом ( γ мю ) А Б . Индексы А и Б являются фермионными индексами или спинорными индексами.
Определение тензора состоит в том, что при преобразовании Лоренца каждый индекс Лоренца сжимается с Λ мю ν и каждый спинорный индекс стягивается с С ( Λ ) А Б . ψ "=" ψ А является контравариантным спинором, и в части 1 упражнения вы по существу показываете, что ψ ¯ "=" ψ ¯ Б является ковариантным спинором. Другой способ подумать о том, что я написал выше, может быть: уравнение γ λ "=" γ λ "=" Λ λ ν ( С γ ν С 1 ) доказывает, что γ является тензором. Показав это, мы можем использовать его в реальном расчете, как это сделал я.
Если это слишком для вас прямо сейчас, вы можете выполнить упражнение, не разбираясь в этом подробно (см. последнюю строку в моем ответе), но я думаю, что это довольно круто, как все это работает :)
Спасибо за вашу помощь. Судя по вашим комментариям, правильно ли я понимаю, что все γ является тензором с двумя спинорными индексами (один контравариантный, один ковариантный) и одним контравариантным «нормальным» индексом? Это действительно довольно круто :) но это слишком новые вещи для меня, чтобы быть уверенным в этих вещах.
Точно. [комментарии должны быть 15 символов]
Можно ли доказать тот факт, что гаммы на самом деле не трансформируются из своих индексов? Если один (спинорный) индекс преобразовывает ковариантно, а другой контравариантно, то преобразования компенсируют друг друга, верно? Но тогда остается контравариантный индекс вектора, значит, они все равно должны преобразовываться..?
Эти три преобразования компенсируют друг друга: индекс контравариантного вектора дает Λ , контравариантный спинорный индекс С и ковариантный спинорный индекс С 1 . Эта комбинация просто отменяется.
Извините, если это нубский вопрос, но не стоит С и С 1 , по определению С 1 , компенсировать друг друга?
Там все еще есть γ в середине: Λ λ ν ( С γ ν С 1 ) "=" γ λ
Есть ли книга, где вы знаете страницу/раздел, где доказано это уравнение? Пескин/Шредер, Вайнберг, Зубер?
Пескин/Шредер: уравнение (3.30). Зи: Согласно уравнению (14), часть 2. Вайнберг: (5.4.8)